LSAT and Law School Admissions Forum

Get expert LSAT preparation and law school admissions advice from PowerScore Test Preparation.

User avatar
 Dave Killoran
PowerScore Staff
  • PowerScore Staff
  • Posts: 5852
  • Joined: Mar 25, 2011
|
#40891
Complete Question Explanation
(The complete setup for this game can be found here: lsat/viewtopic.php?t=15551)

The correct answer choice is (E)

The condition in the question stem initially forms an H :longline: IGK sequence. But, because of the third rule that stipulates that I and L are consecutive, the sequence actually must be H :longline: LIGK. This sequence involves five of the variables, and only F and M are not included. Because the sequence is so large, it has a limited number of placement options:
  • pt59_d09_g2_q7.png
However, option #1 above is impossible because there is no room for M in the first three positions, a violation of the fourth rule. With only options #2 and #3 possible, we could explore each hypothetical and place more of the variables (F, for example). But, a quick glance at the answer choices shows that the two remaining options—even in their basic form—eliminate answer choices (A), (B), (C), and (D) from consideration. Thus, answer choice (E) is proven correct by process of elimination.
You do not have the required permissions to view the files attached to this post.

Get the most out of your LSAT Prep Plus subscription.

Analyze and track your performance with our Testing and Analytics Package.